1
$\begingroup$

Suppose $U$ is a Banach space and let $V\subseteq U$ be bounded, convex and (norm-)closed. Consider the Bochner-Lebesgue space $L^r(0,T;U)$ with $T>0$ and $r\in[1,\infty]$ consisting of strongly measurable functions $u\colon[0,T]\to U$ with $\int_0^T\Vert u(s)\Vert^r_U\,ds<\infty$. For such functions $u$, we define the integral average $w_\varepsilon(s)=(T_\varepsilon u)(s)=\frac1\varepsilon\int_s^{s+\varepsilon}u(t)\,dt$, whenever $s+\varepsilon<T$.

Now, given a net $(u_\iota)_\iota$, weakly$^\ast$ converging in $L^r(0,T;U)$ to some $u$ and being a.e. $V$-valued for any $\iota$, what can we say about convergence of $w_{\varepsilon,\iota}=T_\varepsilon u_\iota$, hopefully against $w_{\varepsilon}=T_\varepsilon u$?

My attempts so far:

  1. Naively inserting and calculating: For any $\varphi\in L^{r/(r-1)}(0,T;U_\#)$ (we view $L^r(0,T;U)$ as a dual space), we have $$ \langle u_\iota-u,\varphi\rangle_{L^r,L^{r/(r-1)}}=\int_0^T\langle (u_\iota-u)(s),\varphi(s)\rangle_{U,U_\#}\,ds\xrightarrow[]{\iota}0 $$ by assumption. Assuming, that $w_{\varepsilon,\iota},w_\varepsilon\in L^r(0,T;U)$ as well, $$ \langle w_{\varepsilon,\iota}-w_\varepsilon,\varphi\rangle_{L^r,L^{r/(r-1)}}=\int_0^T\langle (w_{\varepsilon,\iota}-w_{\varepsilon})(s),\varphi(s)\rangle_{U,U_\#}\,ds=\int_0^T\frac1\varepsilon\int_s^{s+\varepsilon}\langle(u_\iota-u)(t),\varphi(s)\rangle_{U,U_\#}\,dt\,ds, $$ which I so far didn't seem to get working.

  2. Assuming a stronger type of convergence: If $\Vert u_\iota-u\Vert_{L^\infty(0,T;U)}\to0$, then $w_{\varepsilon,\iota}-w_{\varepsilon}\to0$ strongly and weakly$^\ast$. This only covers the case $r=\infty$ and assumes strong convergence, which I don't think I get from my context.

We can adjust $r$, if needed, and perhaps polish my definition of averages a bit. I saw other posts (e.g. Average on the whole space) that relate to my question. Any help is appreciated. Thanks, Joe

$\endgroup$
1
  • $\begingroup$ Couldn't you rewrite the last integral using Fubini theorem as $u_i-u$ integrated against some average of $\phi$? $\endgroup$ Commented Jun 10, 2024 at 14:52

1 Answer 1

1
$\begingroup$

Let me extend the functions with zero from $[0,T]$ to $\mathbb R$.

The idea is to use that the adjoint that the adjoint of the mean value operator is also a mean value operator. Applying Fubini theorem we get $$ \frac1\epsilon\int_{\mathbb R} \int_s^{s+\epsilon} \langle u_i(t) - u(t), \ \phi(s) \rangle dt \ ds = \frac1\epsilon \int_{\mathbb R} \int_{t-\epsilon}^t \langle u_i(t) - u(t), \ \phi(s) \rangle ds \ dt\\ = \int_{\mathbb R} \left \langle u_i(t) - u(t), \ \frac1\epsilon \int_{t-\epsilon}^t\phi(s)\ ds \right\rangle \ dt $$ Now $t\mapsto \frac1\epsilon \int_{t-\epsilon}^t\phi(s)\ ds$ is in $L^{r/(r-1)}(\mathbb R, U_\#)$, so we can pass to the limit in the integral, and obtain convergence to zero.

$\endgroup$
1
  • $\begingroup$ thanks for your help! I also just tried your suggestion of yesterday and got the same. it's also nice to state it in terms of the adjoint of mean value operators, so thanks again also for that! $\endgroup$ Commented Jun 11, 2024 at 8:17

You must log in to answer this question.

Start asking to get answers

Find the answer to your question by asking.

Ask question

Explore related questions

See similar questions with these tags.